Text ch1-2

Download as pdf or txt
Download as pdf or txt
You are on page 1of 86

1

The mathematical sciences particularly exhibit order, symmetry and


limitation; and these are the greatest forms of the beautiful.
– Aristotle

Real Numbers
TOPICS
1. The fundamental theorem of arithmetic.
2. Revisiting irrational numbers.

CONCEPT – 1 : THE FUNDAMENTAL THEOREM OF ARITHMETIC


Introduction: In general there are two types of numbers one prime
numbers and other composite numbers.
Prime Numbers: Numbers which cannot be divided by any number other
than 1 and number itself are known as prime numbers,
for example 2, 3, 5, 7, 11, 13, 17, 19, 23, ...etc.
Composite Numbers: Numbers which can be divided by at least one of the
number other than 1 and itself are known as composite
numbers, for example 4, 6, 8, 9, 10, 12, 14, .... etc.
Prime Factorisation of Composite Numbers
As discussed above that each composite number has at least one divisor other
than one and number itself. So expressing the composite number as the product of all
possible prime numbers is known as the prime factorisation of that composite number.

Note: Prime factorisation of different composite numbers are always different and for each
composite number its prime factors are unique.
Fundamental Theorem of Arithmetic

Statement
Every composite number can be expressed (factorised) as a product of primes,
and this factorisation is unique except for the order in which the prime factors are
written.
For example, prime factors of 30 can be written as:
30 = 2 × 3 × 5 = 2 × 5 × 3 = 3 × 5 × 2 = 3 × 2 × 5 = 5 × 2 × 3 = 5 × 3 × 2
So it is clear that prime factors of 30 can be written in 6 different forms but the
prime numbers occur in the prime factorisation of 30 are same in each case only their
order is different but prime factorisation is unique which is only the conclusion given
by fundamental theorem of Arithmetic.
1
2 S. Chand’s New Mathematics for Class X

Explanation of fundamental theorem of Arithmetic with the help of factor tree

Method to Find HCF and LCM of two or more numbers with the help of Fundamental
Theorem of Arithmetic
Step 1: Find prime factorisation of all the numbers given of which HCF/LCM
is required.
Step 2: For finding HCF take the product of the smallest power of each common
prime factor present in the prime factorisation of given numbers whose
HCF is required.
Step 3: For finding LCM take the product of greatest powers of each prime factor
available in the prime factorisation of given numbers.
Applications of Fundamental Theorem of Arithmetic
1. We can find HCF of two or more numbers.
2. We can find LCM of two or more numbers.
3. If a and b are two distinct numbers and their HCF is ‘H’ and LCM is ‘L’ then
a×b=H×L
H¥L H¥L
⇒ a= or b=
b a
a¥b a¥b
or H= or L=
L H

Concept for finding the unit’s place digit in the given expansion
1. If unit’s place digit of any number say ‘x’ is zero then the units place digit in the
expansion of (x)n when n ∈ N will always be zero.
Or
Real Numbers 3

We can say that expansion of any number (x)n can end with zero only if prime
factors of base i.e., ‘x’ has at least one 2 and one 5 both if one 2 and one 5 is not
available in the prime factors of base i.e., x then expansion of (x)n cannot end with
zero.
2. If unit’s place digit of a number say ‘x’ is 1 then unit’s place digit in the expansion
of (x)n when n ∈ N will always be 1.
3. If unit’s place digit of a number say x is 2 then unit’s place digit in the expansion
of (x)n, n ∈ N will be 2 if n = 1, 4 if n = 2, 8 if n = 3, 6 if n = 4 and it is the rule that
after n = 4 unit’s place digits start repeating as under
For n = 5 unit’s place digit will be same as for n = 1
For n = 6 unit’s place digit will be same as for n = 2
For n = 7 unit’s place digit will be same as for n = 3
For n = 8 unit’s place digit will be same as for n = 4 and this process continues.
4. If unit’s place digit of a number say x is 3 then unit’s place digit in the expansion
of (x)n for n ∈ N will be 3 if n = 1, 9 if n = 2, 7 if n = 3, 1 if n = 4 and after n = 4
repetition of unit’s place digit will start as per rule explained in point number 3
for each value of n = 5 and above.
5. If unit’s place digit of a number say x is 4 then unit’s place digit in the expansion
of (x)n, n ∈ N will be 4 if n = 1, 6 if n = 2, 4 if n = 3, 6 if n = 4 i.e., for each odd ‘n’,
unit’s place digit will be 4 and for each even ‘n’, unit’s place digit will be even.
6. If unit’s place digit of a number say x is 5 then unit’s place digit in the expansion
of (x)n, n ∈ N will always be 5.
7. If unit’s place digit in a number say x is 6, then unit’s place digit in the expansion
of (x)n, n ∈ N will always be 6.
8. If unit’s place digit of a number say x is 7 then unit’s place digit in the expansion
of (x)n, n ∈ N will be 7 if n = 1, 9 if n = 2, 3 if n = 3, 1 if n = 4 and usual repetition
will be applicable for each value of n = 5 and above.
9. If unit’s place digit of a number say x is 8 then unit’s place digit in the expansion
of (x)n, n ∈ N will be 8 if n = 1, 4 if n = 2, 2 if n = 3, 6 if n = 4 and usual repetition
for any value of n = 5 and above.
10. If unit’s place digit of a number say x is 9 then unit’s place digit in the expansion
of (x)n, n ∈ N will be, 9 if n = 1, 1 if n = 2, 9 if n = 3, 1 if n = 4 and so on so we can
conclude in this case that unit’s place digit will be 9 for any odd value of n and 1
for any even value of n.

*Greek symbol ∈ (epsilon) is used to denote the phrase ‘belongs to’.


4 S. Chand’s New Mathematics for Class X

Solved Examples

Type – 1: Examples based on Prime Factorisation


Example 1. Express each number as a product of its prime factors: [NCERT Textbook]
(i) 140 (ii) 156 (iii) 3825 (iv) 5005 (v) 7429
Solution. (i) We use the division method as shown below : 2 140 2 156
\
140 = 2 × 2 × 5 × 7 2 70 2 78
5 35 3 39
= 22 × 5 × 7 7 7 13 13
(ii) We use the division method as shown below : 1 1
\
156 = 2 × 2 × 3 × 13 3 3825 5 5005
3 1275 7 1001
= 22 × 3 × 13
5 425 11 143
(iii) We use the division method as shown below : 5 85 13 13
17 17 1
\
3825 = 3 × 3 × 5 × 5 × 17
1 17 7429
= 32 × 52 × 17 19 437
(iv) We use the division method as shown below : 23 23
1
\
5005 = 5 × 7 × 11 × 13
(v) We use the division method as shown below :
\ 7429 = 17 × 19 × 23
Example 2. Find the LCM and HCF of 6 and 20 by the prime factorisation method.
Solution. We have: 6 = 21 × 31 and 20 = 2 × 2 × 5 = 22 × 51.
HCF (6, 20) = 21 = Product of the smallest power of each common prime
factor in the numbers.
LCM (6, 20) = 22 × 31 × 51 = Product of the greatest power of each prime
factor, involved in the numbers.
From this example we have noticed that HCF (6, 20) × LCM (6, 20) = 6 × 20
we can verify that for any two positive integers a and b,
HCF (a, b) × LCM (a, b) = a × b. We can use this result to find the LCM of
two positive integers, if we already know the two positive integers and
their HCF.

Example 3. Find the missing numbers in the following prime factorisation.


Real Numbers 5

Solution. The product of primes starts from


the bottom of the factor tree and this
product goes up to the top.
The upper box, on 2 and 17 is filled by
the product of 2 and 17, i.e.,  34.
The next upper box, on 2 and 34 is filled
by the product of 2 and 34, i.e., 68.
The topmost box, on 2 and 68 is filled by
the product of 2 and 68, i.e., 136.
Example 4. Find the HCF of 96 and 404 by the prime factorisation method. Hence,
find their LCM.
Solution. The prime factorisation of 96 and 404 are:
96 = 25 × 3, 404 = 22 × 101
Therefore, the HCF of these two integers is 22 = 4.

96 ¥ 404 96 ¥ 404
Also, LCM (96, 404) = = = 9696
HCF (96 , 404) 4

Example 5. Find the HCF and LCM of 6, 72 and 120, using the prime factorisation
method.
Solution. We have,  6 = 2 × 3, 72 = 23 × 32, 120 = 23 × 3 × 5
1 1
Here, 2 and 3 are the smallest powers of the common factors 2 and 3
respectively.
So, HCF (6, 72, 120) = 21 × 31 = 2 × 3 = 6
23, 32 and 51 are the greatest powers of the prime factors 2, 3 and 5
respectively involved in the three numbers.
So, LCM (6, 72, 120) = 23 × 32 × 51 = 360
Remark: Notice, 6 × 72 × 120 ≠ HCF (6, 72, 120) × LCM (6, 72, 120).
So, the product of three numbers is not equal to the product of their
HCF and LCM.

TYPE – 2: Examples based on Unit’s Place Digit of Numbers


Example 6. Find the unit’s place digit in the expansion of the following:
(i) (405)361 (ii) (191)1009 (iii) (67)761 (iv) (84)561
(v) (38)820
Solution. (i) Unit’s place digit in the expansion of (405)361 will be 5 because we
know that the unit’s place digit in the expansion of a number of type :
(x)n, n ∈ N is always 5 if unit’s place digit of the base (x) is 5.
Here, base is 405 its unit’s place digit is 5 so unit’s place digit in the
expansion of (405)361 is 5.
6 S. Chand’s New Mathematics for Class X

(ii) Unit’s place digit in the expansion of (191)1009 will be 1 because we


know that unit’s place digit in the expansion of (x)n, n ∈ N is always
1 if unit’s place digit in the base ‘x’ is 1. Here unit’s place digit of base
of (191)1009 is 1 so unit’s place digit of expansion (191)1009 will be 1.
(iii) Unit’s place digit in the expansion (67)761 will be 7 because we know
that if n = 1 then unit’s digit will be 7.
If n = 2 then unit’s digit will be 9.
If n = 3 then unit’s digit will be 3.
If n = 4 then unit’s digit will be 1.
and for n = 761 we can say 761 = 4 × 190 + 1 i.e., remainder when power
761 is divided by 4 is 1 hence unit’s place digit in the expansion of (67)761
will be same as when n = 1 i.e., 7.
(iv) Unit’s place digit in the expansion of (84)561 will be 4, because
Unit’s place digit in the given expansion will be 4 if n = 1
Unit’s place digit in the given expansion will be 6 if n = 2
Unit’s place digit in the given expansion will be 4 if n = 3
Unit’s place digit in the given expansion will be 6 if n = 4
and so on i.e., for each odd ‘n’ unit’s place digit will be 4.
Hence, unit’s place digit in the expansion of (84)561 will be 4 as value
of n = 561 is odd.
(v) Unit’s place digit in the expansion of (38)820 will be 6, because
Unit’s place digit in the given expansion will be 8 if n = 1
Unit’s place digit in the given expansion will be 4 if n = 2
Unit’s place digit in the given expansion will be 2 if n = 3
Unit’s place digit in the given expansion will be 6 if n = 4
Now, powers of (38) is 820 when divided by 4 remainder is zero.
Hence, unit’s digit of given expansion will be same as in case when
n = 4 i.e., 6.

Example 7. Check whether expansion of (252)n, n ∈ N can end with zero for any n as
natural number.
Solution. We know that the expansion of (x)n, n ∈ N can end with zero only if prime
factorisation of base number i.e., x has at least one 2 and one 5 otherwise
the expansion (x)n can never end with zero.
Now to check whether (252)n can end with zero or not we will find the
prime factors of base i.e., 252 which are as under
252 = 2 × 2 × 3 × 3 × 7 = 22 × 32 × 7
Real Numbers 7

The digit 5 does not appear in the prime factors of 252.


Hence, (252)n can never end with zero for any n ∈ N.
Example 8. Check whether (75)n can end with zero for any n ∈ N.
Solution. No, because prime factors of base i.e., 75 are 3 × 5 × 5. There is no. 2 in
the prime factors of 75 hence for any n ∈ N (75)n cannot end with zero.
Example 9. Prove that there is no natural number for which 4n ends with the digit
zero.
Solution. We know that any positive integer ending with the digit zero is divisible
by 5 and so its prime factorisation must contain the prime 5.
We have,
n
4n = (22) = 22n
⇒ The only prime in the factorisation of 4n is 2.
⇒ There is no other primes in the factorisation of 4n = 22n
[By uniqueness of the Fundamental Theorem of Arithmetic]
⇒ 5 does not occur in the prime factorisation of 4n for any n.
⇒ 4n does not end with the digit zero for any natural number n.
Example 10. Check whether 6n can end with the digit 0 or any n ∈ N. [NCERT Textbook]
Solution. If the number 6n ends with the digit zero. Then it is divisible by 5. Therefore
the prime factorisation of 6n contains the prime 5. This is not possible
because the only primes in the factorisation of 6n are 2 and 3 and the
uniqueness of the fundamental theorem of arithmetic guarantees that
there are no other prime in the factorisation of 6n.
So, there is no value of n in natural numbers for which 6n ends with the
digit zero.

TYPE – 3: Examples based on Application of Fundamental Theorem of Arithmetic

Example 11. Explain why 7 × 11 × 13 + 13 and 7 × 6 × 5 × 4 × 3 × 2 × 1 + 5 are composite


numbers. [NCERT Textbook]
Solution. We have, 7 × 11 × 13 + 13 = 1001 + 13 = 1014
1014 = 2 × 3 × 13 × 13
So, it is the product of more than two prime numbers.
2, 3 and 13.
Hence, it is a composite number.
7 × 6 × 5 × 4 × 3 × 2 × 1 + 5 = 5040 + 5 = 5045
5045 = 5 × 1009
It is the product of prime factors 5 and 1009.
Hence, it is a composite number. Ans.
8 S. Chand’s New Mathematics for Class X

Example 12. There is a circular path around a sports field. Sonia takes 18 minutes to
drive one round of the field, while Ravi takes 12 minutes for the same.
Suppose they both start at the same point and at the same time, and go
in the same direction. After how many minutes will they meet again at
the starting point? [NCERT Textbook]
Solution. They will be again at the starting point at least at common multiples of
18 and 12 minutes. To find the LCM of 18 and 12, we have
2 18 2 12
18 = 2 × 3 × 3 and 12 = 2 × 2 × 3
3 9 2 6
LCM of 18 and 12 = 2 × 2 × 3 × 3 = 36 3 3 3 3
1 1
So, Sonia and Ravi will meet again at the starting point after
36 minutes.
Example 13. A circular field has a circumference of 360 km. Two cyclists Aayush and
Anuj start together and can cycle at speeds of 12 km/hr and 15 km/hr,
respectively, round the circular field. After how many hours will they
meet again at starting point?
Solution. Speed of Aayush = 12 km/hr.
360
Number of hours taken by Aayush to complete one round = = 30
12
Speed of Anuj = 15 km/hr
360
Number of hours taken by Anuj to complete one round = = 24
15

So, Aayush and Anuj complete one round in 30 hrs and 24 hrs, respectively.
Now, let us find the LCM of 30 and 24.
30 = 2 × 3 × 5
24 = 23 × 3
Then, LCM (30, 24) = 23 × 3 × 5 = 120.
Hence, Aayush and Anuj will meet each other again after 120 hours.
Example 14. Four bells toll at an interval of 8, 12, 15 and 18 seconds respectively.
All the four bells begin to toll together. How many times will they toll
together in one hour excluding the one at the start?
Solution. LCM of 8,12,15 and 18 = 360
So, after each 360 seconds i.e., after each 6 minutes four bells will toll
together so in 1 hour i.e., in 60 minutes these four bells will toll together
60
= = 10 times
6
Real Numbers 9

Example 15. In a school there are two sections of class X and the sections are A and B.
There are 96 students in section A and 120 students in section B. Determine
the least number of books required for the library of the school so that
the books can be distributed equally among students of the section A or
that of the section B?
Solution. Least number of books required = LCM of 96 and 120
96 = 2 × 2 × 2 × 2 × 2 × 3 = 25 × 3
120 = 2 × 2 × 2 × 3 × 5 = 23 × 31 × 51
Hence, LCM (96, 120) = 25 × 31 × 51 = 480.
So, required number of books = 480.

Exercise 1.1
TYPE – 1: Questions based on Prime Factorisation
1. Find the prime factors of the following numbers:
(i) 176 (ii) 256 (iii) 4825 (iv) 12673
2. Find the missing numbers in the following factor trees.
(i) (ii)

3. Find the LCM and HCF of the following pairs of integers by applying the
Fundamental Theorem of Arithmetic method.
(i) 455, 78 (ii) 408, 170 (iii) 13, 11
4. Find the LCM and HCF of the following integers by applying the prime factorisation
method.
(i) 275, 225, 175 (ii) 765, 510, 408 (iii) 19, 13, 7
5. Given that HCF (1261, 1067) = 97, find LCM (1261, 1067). [CBSE 2020]
6. Find the LCM and HCF of the following pairs of integers and verify:
LCM × HCF = Product of the two numbers.
(i) 26 and 91 (ii) 510 and 92 (iii) 336 and 54

TYPE – 2: Questions based on Unit’s Place Digit


7. Check whether (15)n can end with the digit 0 for any n ∈ N.
8. Check whether (28)n can end with the digit 0 for any n ∈ N.
9. Check whether (26)n can end with the digit 5 for any n ∈ N.
10. Check whether (2)n can end with the digit 6 for any n ∈ N.
11. Check whether (3)n can end with the digit 1 for any n ∈ N.
10 S. Chand’s New Mathematics for Class X

Type – 3: Questions based on Application of Fundamental Theorem of Arithmetic


12. Explain why: 3 × 5 × 7 + 7 × 11 is a composite number.
13. Explain why: 5 × 7 × 11 + 13 × 17 is a composite number. Also find smallest divisor.
14. Check whether following numbers are prime or composite
(i) 2 × 3 + 5 (ii) 7 × 11 × 13 + 13 (iii) 3 × 7 + 2
(iv) 13 × 17 × 19 + 23 × 38
15. A rectangular field is 150 m × 60 m. Two cyclists Karan and Vijay start together and
can cycle at speed of 21 m/min and 28 m/min, respectively. They cycle along the
rectangular track, around the field from the same point and at the same moment.
After how many minutes will they meet again at the starting point?
16. Radius of a circular track is 63 m. Two cyclists Amit and Ajit start together
from the same position, at the same time and in the same direction with speeds
33 m/min and 44 m/min. After how many minutes they meet again at the starting
point?
17. On a morning walk, three persons step off together and their steps measure
40 cm, 42 cm and 45 cm, respectively. What is the minimum distance each should
walk so that each can cover the same distance in complete steps?
18. An army contingent of 455 members is to march behind an army band of
42 members in a parade. The two groups are to march in the same number of
columns. What is the maximum number of columns in which they can march?
19. Three sets of English, Hindi and Mathematics books have to be stacked in such
a way that all the books are stored topic wise and the height of each stack is the
same. The number of English books is 96, the number of Hindi books is 240 and
the number of Mathematics books is 336. Assuming that the books are of the same
thickness, determine the number of stacks of English, Hindi and Mathematics
books.
20. In a delegation of students there are 60 Indian, 84 Chinese and 108 British. They all
want to stay in the same hotel. The number of students stay in each room is same
and will also stay with their own country students. Find the minimum number
of rooms required for stay of these students.

ANSWERS
1. (i) 176 = 2 × 2 × 2 × 2 × 11 (ii) 256 = 2 × 2 × 2 × 2 × 2 × 2 × 2 × 2
(iii) 4825 = 5 × 5 × 1931 (iv) 12673 = 19 × 23 × 29
2. (i) From top to bottom 15015, 5005, 7, 13
(ii) From top to bottom 3, 5, 1729, 247
3. (i) HCF = 13, LCM = 2730 (ii) HCF = 34, LCM = 2040
(iii) HCF = 1, LCM = 143
4. (i) HCF = 25, LCM = 17325, (ii) HCF = 51, LCM = 6120
(iii) HCF = 1, LCM = 1729
5. LCM = 13871 6. (i) LCM = 182, HCF = 13, (ii) LCM = 23460, HCF = 2,
(iii) LCM = 3024, HCF = 6
Real Numbers 11

7. No 8. No 9. No
10. Yes for n = 4 11. Yes for n = 4
12. Yes 13. Yes
14. (i) Prime   (ii) Composite  (iii) Prime  (iv) Composite
15. 60 minutes 16. 36 minutes 17. (2520 cm)
18. 7 members 19. English = 2, Hindi = 5, Mathematics = 7
20. No. of rooms = 21.

Concept – 2 : REVISITING IRRATIONAL NUMBERS


Definition
p
1. A number which cannot be expressed in the form of [where p and q are
q
co-primes (relatively - prime)] and q ≠ 0 is known as irrational numbers.
2. A number which cannot be expressed either in terminating form or non-terminating
repeating form then it is known as irrational number.
3. Numbers in the form of non-terminating non-repeating are known as irrational
numbers.
1
For example, 2 , 3 , , 3.1020020002 ..........
2

Note: As we know that sum, difference, product and division of any two rational numbers
is always rational (for division rational number in denominator should not be zero).
Similarly, sum, difference, product and division of a rational number with an irrational
always results in an irrational number.
Theorem
Let p be a prime number. If p divides a2, then p divides a, where a is a positive
integer.
Proof. Let p1, p2, p3 ... pn be the factors of a.
⇒ a = p1. p2. p3 ... pn, where p1, p2, .... are primes not necessarily all distinct.
Therefore, a2 = (p1. p2. p3 ... pn) . (p1. p2. p3 .... pn)
⇒ a2 = p12. p22. p32 ... pn2
Give that, p divides a2 ... (1)
2
⇒ p is a prime factor of a , from the Fundamental Theorem of Arithmetic.
the prime factors of a2 are only p1, p2, p3 .... pn ... (2)
(Uniqueness of factors of Fundamental Theorem of Arithmetic)
From (1) and (2), we have
p is a prime factor of p1. p2. p3 .... pn
⇒ p divides a. Proved.
12 S. Chand’s New Mathematics for Class X

Solved Examples

Type – 1: Revisiting Irrational Numbers


Example 16. Prove that 3 is an irrational number. (CBSE 2020)
Solution. To prove ÷3 is an irrational number we will use contradiction method
i.e., first we will assume the opposite of that what we want to prove and
then at the end we will prove that what we assume is wrong and finally
we will come in a position to prove the required result.
Now, if possible let ÷3 is a rational number so as per definition of rational
p
numbers we can say that 3 =
q
{where p and q are co-primes and q ≠ 0} ...(1)
2
2 Ê pˆ
On squaring both the sides of equation (1) we get ( 3 ) = Á ˜
Ë q¯
p2
⇒ 3= 2 ⇒ p2 = 3q2 ...(2)
q
It is very clear that RHS of equation (2) is 3q2, so we can say this result
(3q2) is clear cut multiple of 3 [as 3q2 = 3 × q2]
As RHS of the equation (2) is a multiple of 3. So LHS i.e., p2 will also be
multiple of 3, because LHS and RHS of any equation are same [equal]
from above discussion it becomes clear that p2 is multiple of 3.
So, p is multiple of 3. ...(3)
From equation (3) we are saying that p is multiple of 3 so we can say that
p = 3m [where m is any integer]
Now, on replacing ‘p’ by 3m in equation (2) we get
(3m)2 = 3q2
⇒ 3m × 3 m = 3q2
⇒ 3m2 = q2 ...(4)
Now, clearly LHS of equation (4) is a multiple of 3 because LHS is 3 m2
which can further be written as 3 × m2.
So, RHS, i.e., ‘q2’ of equation (4) will also be a multiple of 3 but q2 can be
a multiple of 3 only if q is a multiple of 3. ...(5)
Now, from equations (3) and (5) we can conclude that ‘p’ and ‘q’ both are
a multiple of 3.
But in the beginning in equation (1) we have assumed [as per definition of
rational numbers] that p and q are co-prime i.e., p and q have no common
factor other than 1.
Real Numbers 13

But right now we have shown that ‘p’ and ‘q’ have 3 as common factor
so it contradicts our supposition that ÷3 is rational.
Hence, our supposition that ÷3 is rational is wrong.
Hence, we can conclude from this that ÷3 is an irrational number.

Example 17. Prove that ÷2 is an irrational number by contradiction method.


Solution. We assume that ÷2 is rational. (CBSE 2019)
p
⇒ 2= (where p and q are co-primes, q ≠ 0)
q
p2
⇒ 2= (Squaring both sides)
q2
⇒ p2 = 2q2 ...(1)
2 2 2
⇒ 2q = p ⇒ 2 divides p
⇒ 2 divides p ...(2) (By Concept 2)
2 2
Let p = 2m ⇒ p = 4m
2
Putting the value of p in (1), we have
4m2 = 2q2 ⇒ 2m2 = q2
⇒ 2 divides q2 ⇒ 2 divides q (By Concept 2)
Thus, 2 divides p and q [From (2)]
It means 2 is a common factor of p and q. This contradicts the assumption
as there is no common factor of p and q.
Thus, ÷2p is not rational.
Hence, ÷2 is irrational. Proved.

Example 18. Prove that ÷5 is an irrational number by contradiction method.


(NCERT Textbook)
Solution. Suppose ÷5 represents a rational number. Then ÷5 can be expressed
p
in the form , where p,  q are integers and have no common factor,
q
q ≠ 0.
p
5=
q
Squaring both sides, we get
p2
5= ⇒ p2 = 5q2 ...(1)
q2
⇒ 5 divides p2
⇒ 5 divides p. ... (2) (Concept 2)
14 S. Chand’s New Mathematics for Class X

Let p=5m ⇒ p2 = 25 m2
Putting the value of p2 in (1), we get
25 m2 = 5 q2 ⇒ 5 m2 = q2
⇒ 5 divides q2 ⇒ 5 divides q. ... (3) (Concept 2)
Thus, from (2), 5 divides p and from (3), 5 also divides q. It means 5 is a
common factor of p and q. This contradicts the supposition so there is no
common factor of p and q.
Hence, ÷5 is an irrational number. Proved.
Example 19. Show that ÷7 is an irrational number. (Withous using contradiction
method)
Solution. Let us find out square root of 7 by division method.
2 . 6 4 5 7 5 1 3
2 7.00 00 00 00 00 00 00
2 –4
46 300
6 –276
524 2400
4 –2096
5285 30400
5 –26425
5 2 9 0 7 397500
7 –370349
5 2 9 1 4 5 2715100
5 –2645725
5 2 9 1 5 0 1 6937500
1 –5291501
5 2 9 1 5 0 2 3 1645999

⇒ ÷7 = 2.6457513...... is non-terminating and non-recurring decimal.


Hence, ÷7 is an irrational number. Proved.
Example 20. Prove that ÷7 is an irrational number by contradiction method.
Solution. Suppose ÷7 represents a rational number. Then ÷7 can be expressed in
p
, where p, q are co-primes (q ≠ 0).
the form
q
p
7=
q
Squaring both the sides, we get
p2
7= ⇒ 7q2 = p2 ... (1)
q2
Real Numbers 15

⇒ 7 divides p2 ⇒ 7 divides p ... (2) (Concept 2)


2 2
Let p=7m ⇒ p = 49 m
Putting the value of p2 in (1), we get
7q2 = 49 m2 ⇒ q2 = 7m2
⇒ 7 divides q2 ⇒ 7 divides q ... (3) (Concept 2)
Thus, from (2), p is a multiple of 7 and from (3), q is also a multiple of 7.
It means 7 is a common factor of p and q. This contradicts the supposition
so there is no common factor of p and q.
Hence, ÷7 is an irrational number. Proved.
Example 21. Prove that 3 + 2÷5 is irrational. (CBSE 2020, NCERT Textbook)
Solution. Let us assume, to the contrary, that 3 + 2÷5 is a rational number.
a
Now, let 3 + 2 5 = , where a and b are co-primes and b ≠ 0.
b
a a 3
So, 2÷5 = – 3 or 5= -
b 2b 2
Since, a and b are integers, therefore
a 3
- is a rational number
2b 2
\ ÷5 is a rational number.
But ÷5 is an irrational number.
This shows that our assumption is incorrect.
So, 3 + 2÷5 is an irrational number. Proved.
Example 22. Prove that the following are irrationals:
1
(i) (ii) 7÷5 (iii) 6 + ÷2 (NCERT Textbook)
2
1
Solution. (i) Let us assume, to the contrary, that is rational. That is, we can
2
find co-prime integers p and q (q ≠ 0) such that
1 p 1¥ 2 p 2 p
= or = or =
2 q 2¥ 2 q 2 q
2p
or ÷2 =
q
2p
Since, p and q are integers so is rational, and so ÷2 is rational.
q
But this contradicts the fact that ÷2 is irrational.
So, we conclude that ÷2 is an irrational. Proved.
16 S. Chand’s New Mathematics for Class X

(ii) Let us assume, to the contrary, that 7÷5 is rational. That is, we can
p
find co-prime integers p and q (q ≠ 0) such that 7 5 = .
q
p
So, 5= .
7q
p
Since, p and q are integers, is rational and so is ÷5.
7q
But this contradicts the fact that ÷5 is irrational.
So, we conclude that 7÷5 is an irrational. Proved.

(iii) Let us assume, to the contrary, that 6 + ÷2 is rational. That is, we


can find integers p and q (≠ 0) such that
p p
6 + ÷2 = or 6 - = 2
q q
p
or ÷2 = 6 -
q
p
Since, p and q are integers, we get 6 - is rational, and so ÷2 is rational.
q
But this contradicts the fact that ÷2 is irrational.
So, we conclude that 6 + ÷2 is an irrational. Proved.
3
Example 23. Show that 6 is not a rational number.
3
Solution. Let us assume that 6 is a rational number.

3 p
Let 6 = , (where p and q are co-primes, q ≠ 0)
q
p3
=6 ... (1) (Taking cube on both sides)
q3
Now 13 = 1 and 23 = 8

p3 p
Thus, 1<6<8 ⇒ 1< 3
< 8 fi 1< <2
q q
3
p p3
From (1), =6 fi 6q 2 = ...(2)
q3 q
As q is an integer ⇒ 6q2 is an integer ...(3)
Since, p and q have no common factor.
⇒ p3 and q will have no common factor.

p3
⇒ is a fraction ...(4)
q
Real Numbers 17

From (2), (3) and (4)


Integer = fraction
Thus, our assumption is wrong.
3
Hence, 6 is not a rational number. Proved.

Example 24. Prove that n is not a rational number, if n is not a perfect square.
Solution. n is a rational number.
Suppose
p
n=
q
(p and q have no common factor co-primes, and q ≠ 0)
p2
⇒ n= (Squaring both sides)
q2
⇒ p2 = nq2 ...(1)
2
⇒ n divides p
⇒ n divides p ... (2) (Concept 2)
Let p = nm
⇒   p2 = n2 m2 (Squaring both sides)
2
Putting the value of p in (1), we get
n2m2 = nq2
⇒ q2 = nm2
⇒ n divides q2
⇒ n divides q ... (3) (Concept 2)
From (2), n divides p and from (3) n divides q. It means n is a factor of
both p and q.
This contradicts the assumption that p and q have no common factor.
So our supposition is wrong. Hence, ÷n cannot be a rational number.
Proved.
Example 25. Prove that ÷2 + ÷3 is an irrational number. (NCERT Textbook)
Solution. Assume that ÷2 + ÷3 is a rational number ‘a’.
\ a= 2+ 3
⇒ a 2 = ( 2 + 3 )2 (Squaring both sides)
2
⇒ a = 2+3+2 2 3
⇒ a2 = 5 + 2 6
⇒ a2 - 5 = 2 6
a2 - 5
⇒ = 6 ...(1)
2
18 S. Chand’s New Mathematics for Class X

a2 - 5
Now is a rational number as ‘a’ is a rational number (by assumption)
2
From (1), ÷6 is a rational number. [\ LHS is rational so RHS will be rational]
But ÷6 is an irrational number. (Example 24)
Thus, there is a contradiction.
Hence, our assumption is wrong.
Therefore, ÷2 + ÷3 is an irrational number. Proved.

Exercise 1.2
1. Prove that ÷5 is an irrational number.
2. Prove that 2 + ÷6 is an irrational number.
3. Prove that ÷7 – 8 is an irrational number.
4. Prove that 3÷11 is an irrational number.
5
5. Prove that is an irrational number.
2
6. Prove that (÷3 – ÷8) is an irrational number.
7. Prove that ÷13 + ÷17 is an irrational number.
8. Prove that 3÷10 is an irrational number.
9. Give rational approximation of ÷5 correct to two decimal places.
10. Identify the following as rational or irrational.
9
(i) ÷4 (ii) 3÷18 (iii) ÷1.44 (iv)
27
(v) –÷0.64 (vi) ÷100 (vii) (÷3 + 1)2 (viii) (÷5 + 1)(÷5 – 1)
6
(ix) ( 3 + 4 )( 5 + 6 ) (x)
2 3

ANSWERS
9. 2.23
10. (i) Rational Number (ii) Irrational Number
(iii) Rational Number (iv) Irrational Number
(v) Rational Number (vi) Rational Number
(vii) Irrational Number (viii) Rational Number
(ix) Irrational Number (x) Irrational Number.
Real Numbers 19

REVISION EXERCISE
section – a

Multiple Choice Questions (MCQs) (1 Mark each)

1. The [HCF × LCM] for the numbers 50 and 20 is


(a) 10 (b) 100 (c) 1000 (d) 50 Ans. (c)
If least prime factor of ‘a’ is 3 and least prime factor of ‘b’ is 7, the least prime
2. 
factor of (a + b) is:
(a) 2 (b) 3 (c) 5 (d) 11 Ans. (a)
3. If a, b are co-prime, then a2, b2 are :
(a) Co-prime (b) Not co-prime
(c) Odd numbers (d) Even numbers Ans. (a)
2 2
4. 119 – 111 is:
(a) Prime number (b) Composite number
(c) An odd prime number (d) An odd composite number Ans. (b)
5. Numbers of the form (41)n, n ∈ N will always end with the digit.
(a) zero (b) 2 (c) 3 (d) 1 Ans. (d)
6. Which of the following is not an irrational number?
(a) 5 - 3 (b) 5+ 3 (c) 4+ 2 (d) 5 + 9 Ans. (d)
7. Which is not an Irrational number?
(a) 5 – ÷3 (b) 2+ 5 (c) 4 + ÷2 (d) 6 + ÷9 Ans. (d)
2 2 2
8. If two positive integers a and b are written as a = x y and b = xy ; x, y are prime
numbers then HCF (a, b) is:
(a) xy (b) xy2 (c) x2y3 (d) x2y2 Ans. (b)
3 2 2 3
9. If x = 2 × 3 × 5 , y = 2 × 3 , then HCF (x, y) is:
(a) 12 (b) 108 (c) 6 (d) 36 Ans. (a)
10. Given that HCF (2520, 6600) = 120, LCM (2520, 6600) = 252 × k, then the value
of k is:
(a) 550 (b) 1600 (c) 165 (d) 1625 Ans. (a)
11. If p, q are two co-prime numbers, then HCF (p, q) is:
(a) p (b) q (c) pq (d) 1 Ans. (d)
12. The product of the HCF and LCM of the smallest prime number and smallest
composite number is:
(a) 2 (b) 4 (c) 6 (d) 8 Ans. (d)
20 S. Chand’s New Mathematics for Class X

13. If p, q are two prime numbers, then LCM (p, q) is:


(a) 1 (b) P (c) q (d) pq Ans. (d)
14. If d = HCF (48, 72), the value of d is:
(a) 24 (b) 48 (c) 12 (d) 72 Ans. (a)
15. Given that LCM (91, 26) = 182, then HCF (91, 26) is:
(a) 13 (b) 26 (c) 7 (d) 9 Ans. (a)
16. If d = LCM (36, 198), then the value of d is:
(a) 396 (b) 198 (c) 36 (d) 1 Ans. (a)
17. Given that HCF (26, 91) = 13, then LCM (26, 91) is:
(a) 2366 (b) 182 (c) 91 (d) 364 Ans. (b)
18. (2 + ÷5) (2 – ÷5) expression is:
(a) A rational number (b) A whole number
(c) An integer (d) All of these Ans. (d)
19. (
2- 3 )(
3 + 2 is )
(a) A rational number (b) A whole number
(c) An irrational number (d) A natural number Ans. (a)
20. Given that HCF (253, 440) = 11 and LCM (253, 440) = 253 × R . The value of R is:
(a) 400 (b) 40 (c) 440 (d) 253 Ans. (b)
21. If n is any natural number, then which of the following expressions end with 0:
(a) (3 × 2)n (b) (4 × 3)n (c) (2 × 5)n (d) (6 × 2)n Ans. (c)
22. How many prime factors are there in prime factorization of 5005?
(a) 2 (b) 4 (c) 6 (d) 7 Ans. (b)

section – B
Very Short Answer Type Questions (2 Marks each)
1. Find the LCM of (306, 1314), m

if HCF of (306, 1314) = 18.


Ans. 22338 2
m
2. Find the LCM of 336 and
54 by prime factorisation
method. Ans. 3024 2 n 2

3. The HCF and LCM of


two numbers are 9 and 90
2 3 n
respectively. If one number
is 18, find the other. Ans. 45
4. In the adjoining factor tree (I), 2 5
find the numbers m and n:
Ans. m = 160, n = 40
2 5 5 2
5. Find the LCM and HCF of
120 and 144 by fundamental
(I) (II)
theorem of Arithmetic. Ans. HCF = 24, LCM = 720
Real Numbers 21

6. In the adjoining factor tree (II), find the numbers m and n.


Ans. m = 300, n = 50

section – C
Short Answer Type Questions (3 Marks each)
1. Prove that 3 + ÷7 is an irrational number.
2. Prove that 2÷3 – 4 is an irrational number.
3. A merchant has 120 litres of oil of one kind, 180 litres of another kind and
240 litres of third kind. He wants to sell the oil by filling the three kinds of oil in
tins of equal capacity. What should be the greatest capacity of such a tin?
Ans. 60 litres
4. Find the HCF and LCM of 306 and 54. Verify that HCF × LCM = Product of the
two numbers. Ans. HCF = 18, LCM = 918
5. Find the LCM and HCF of 15, 18, 45 by the prime factorisation method.
Ans. LCM = 90, HCF = 3
1
6. Prove that is an irrational number.
2+ 3
7. Show that 9n cannot end with 2 for any integer n.
8. Prove that 3 + 5 is an irrational number.
9. Prove that 2 3 - 7 is an irrational number.
10. Prove that (5 - 3 ) is an irrational number.

SOLUTIONs FOR NCERT TEXTBOOK EXERCISES N


C
NCERT Exercise 1.1
E
1. Express each number as a product of its prime factors: R
(i) 140 (ii) 156 (iii) 3825 (iv) 5005 (v) 7429 T
Sol. Page 4 Example 1
2. Find the LCM and HCF of the following pairs of integers and verify that T
LCM × HCF = Product of the two numbers. e
(i) 26 and 91 (ii) 510 and 92 (iii) 336 and 54 x
Sol. (i) Prime factors of 26 = 2 × 13
t
b
Prime factors of 91 = 7 × 13
o
Hence, HCF = Common factors between 26 and 91 o
= 13 and LCM = 13 × 2 × 7 = 182 k
Now product of numbers 26 and 91
    = 26 × 91 = 2366 and Product of HCF and LCM = 13 × 182 = 2366
So, verify that product of two numbers = Product of HCF and LCM.
22 S. Chand’s New Mathematics for Class X

(ii) Prime factors of 510 = 2 × 3 × 5 × 17


Prime factors of 92 = 2 × 2 × 23
Hence, HCF = 2 and LCM = 2 × 2 × 3 × 5 × 17 × 23 = 23460
Now product of numbers 510 and 92 = 46920
and product of HCF and LCM = 2 × 23460 = 46920
Hence, verified that product of two numbers is equal to product of their HCF
and LCM.
(iii) Prime factors of 336 = 2 × 2 × 2 × 2 × 3 × 7
Prime factors of 54 = 2 × 3 × 3 × 3
Hence, HCF (Product of common factors of 336 and 54)
=2×3=6
and LCM (Product of all common factors with remaining factors)
= (2 × 3) × 2 × 2 × 2 × 3 × 3 × 7 = 3024
Now, product of numbers 336 and 54 = 336 × 54 = 18144
and product of HCF and LCM = 6 × 3024 = 18144
Hence, product of two numbers = Product of HCF and LCM.
3. Find the LCM and HCF of the following integers by applying the prime factorisation
method.
(i) 12, 15 and 21 (ii) 17, 23 and 29 (iii) 8, 9 and 25
Sol. (i) Prime factors of 12 = 2 × 2 × 3
Prime factors of 15 = 3 × 5
Prime factors of 21 = 3 × 7
So, HCF (Product of all common factors of given three numbers 12, 15
and 21) = 3 and LCM of 12, 15 and 21 = Product of common factors with other
N remaining factors i.e., LCM = 3 × 22 × 5 × 7 = 420
C (ii) Prime factors of 17 = 17 × 1
E Prime factors of 23 = 23 × 1
R Prime factors of 29 = 29 × 1
T So, HCF of 17, 23 and 29 = 1 and their LCM = 17 × 23 × 29 = 11339
(iii) Prime factors of 8 = 2 × 2 × 2
T Prime factors of 9 = 3 × 3
e
Prime factors of 25 = 5 × 5
x
So, HCF of 8, 9 and 25 is 1 because there is no common factors in the prime
t factorisation of 8, 9 and 25.
b Now LCM of 8, 9 and 25 = 8 × 9 × 25 = 1800
o 4. Given that HCF (306, 657) = 9, find LCM of (306, 657).
o Sol. As given HCF of 306 and 657 is 9, so we can find LCM by using rule
k HCF × LCM = Product of two numbers
⇒ 9 × LCM = 306 × 657
306 ¥ 657
⇒ LCM =
9
Real Numbers 23

⇒ LCM = 22338
n
5. Check whether 6 can end with the digit 0 for any natural number n.
Sol. Page 7 Example 10
N
6. Explain why 7 × 11 × 13 + 13 and 7 × 6 × 5 × 4 × 3 × 2 × 1 + 5 are composite numbers.
C
Sol. Page 7 Example 11
E
7. There is a circular path around a sports field. Sonia takes 18 minutes to drive one
R
round of the field, while Ravi takes 12 minutes for the same. Suppose they both T
start at the same point and at the same time, and go in the same direction. After
how many minutes will they meet again at the starting point? e
Sol. Page 8 Example 12 x
NCERT Exercise 1.2
e
m
1. Prove that ÷5 is irrational. p
Sol. Page 13 Example 18 l
2. Prove that 3 + 2÷5 is irrational. a
Sol. Page 15 Example 21 r
3. Prove that following are irrationals
1
(i) (ii) 7÷5 (iii) 6 + ÷2
2
Sol. Page 15 Example 22
SOLUTIONs FOR NCERT exemplar EXERCISES
NCERT exemplar Exercise 1.1
5. The largest number which divides 70 and 125, leaving remainder 5 and 8
respectively  is:
(a) 13 (b) 65 (c) 875 (d) 1750
6. (b) If two positive integers ‘a’ and ‘b’ are written as a = x3y2 and b = xy3, where x, y are
prime numbers. HCF of ‘a’ and ‘b’ is:
(a) xy (b) xy2 (c) x3y3 (d) x2y2
3 2
Sol. Prime factors of a = x y = x × x × x × y × y
Prime factors of b = xy3 = x × y × y × y
So, HCF = Product of common factors = xy2
Hence, option (b) is correct.
7. For two positive integers p = ab2 and q = a3b LCM is [a and b are prime numbers]
(a) ab (b) a2b2 (c) a3b2 (d) a3b3
Sol. (c) Clearly p = a × b × b and q = a × a × a × b
So, LCM will be a3 × b2
Hence, option (c) is correct.
8. The product of a non-zero rational nuimber and an irrational number is:
(a) always irrational (b) always rational
(c) rational or irrational (d) one
Sol. (a) It is true.
24 S. Chand’s New Mathematics for Class X

9. The least number that is divisible by all the numbers from 1 to 10 (both inclusive)
is:
N (a) 10 (b) 100 (c) 504 (d) 2520
C Sol. (d) Clearly such least number is LCM of 1, 2, 3, 4, 5, 6, 7, 8, 9 and 10.
E Which is 2520.
R Hence, option (d) is correct.
T
NCERT exemplar Exercise 1.2
e 6. The numbers 525 and 3000 are both divisible only by 3, 5, 15, 25 and 75. What is
x HCF of (525, 3000)? Justify your answer.
e Sol. Since, the HCF (525, 3000) = 75
m and the numbers 3, 5, 15, 25 and 75 divides the numbers 525 and 3000 that mean
these terms are common in both 525 and 3000. So, the highest common factor
p among these is 75.
l 7. Explain why 3 × 5 × 7 + 7 is composite number.
a Sol. Value of 3 × 5 × 7 + 7 = 112
r 112 can be written as (112) = 24 × 7
As 112 can be expressed as product of two or more prime number hence,
3 × 5 × 7 + 7 is composite number.
8. Can two numbers have 18 as their HCF and 380 as their LCM? Give reason.
Sol. No, because 380 is not divisible by 18.
Rule: LCM is always divisible by HCF.
NCERT exemplar Exercise 1.3
10. Prove that ÷3 + ÷5 is irrational.
Sol. If possible let ÷3 + ÷5 is rational. So we can say ÷3 + ÷5 = r {where ‘r’ is rational
number}
As ÷3 + ÷5 = r ... (1)
On squaring both sides of equation (1)
We get 8 + 2÷15 = r2

r2 - 8
⇒ ÷15 = ... (2)
2
Clearly RHS of equation (2) is rational and LHS is irrational.
So, our supposition is wrong.
Hence, ÷3 + ÷5 is irrational.
11. Show that (12)n cannot end with digit ‘0’ or ‘5’ for any natural number ‘n’.
(CBSE 2020)
Sol. For a number to end with digit zero its prime factors must have atleast one 2 and
one 5 both and for a number to end with 5 its prime factors must have at least
one  5.
Real Numbers 25

But in case of (12)n there is no ‘5’ in prime factors of 12 i.e., 12 = 2 × 2 × 3.


Hence (12)n can neither end with zero nor by 5.
12. On a morning walk, three persons step off together and their steps measure 40 cm,
N
42 cm and 45 cm, respectively. What is the minimum distance each should walk C
so that each can cover the same distance in complete steps? E
Sol. We have to find the LCM of 40 cm, 42 cm and 45 cm to get the required minimum R
distance. T
For this, 40 = 2 × 2 × 2 × 5
42 = 2 × 3 × 7 e
and 45 = 3 × 3 × 5 x
LCM (40, 42, 45) = 2 × 3 × 5 × 2 × 2 × 3 × 7 e
= 30 × 12 × 7 = 210 × 12 m
= 2520 p
Minimum distance each should walk 2520 cm. So that, each can cover the same l
distance in complete steps. a
14. Prove that p + q is irrational, where p, q are primes. r
Sol. Let us suppose that p + q is rational.

Again, let p + q = a, where a is rational.

Therefore, q =a– p
On squaring both sides, we get
q = a2 + p – 2a p [Q (a – b)2 = a2 + b2 – 2ab]
a2 + p - q
Therefore, p = , which is a contraction as the right-hand side is a
2a
rational number while p is irrational, since p and q are prime numbers.

Hence, p + q is irrational.

CASE STUDIES
Case Study 1
Resident welfare association
of a town decided to create
a library for the children of
their locality to enhance the
How do we know what
interest of children toward number to look for?
studies and learning moral
values. They put 12 books of
Vedic Mathematics, 18 books
of Social Studies, 30 books of
General Science and 42 Books
of Moral Education.
26 S. Chand’s New Mathematics for Class X

In the library at one time if equal number of children comes to read the books of Vedic
Mathematics, Social Studies, General Science and Moral Education at same time as
decided by R.W.A. All these books are of same size and will be kept in minimum
number of shelves in the library so that each shelf contains equal number of books and
of same subject.
1. What is the maximum number of students of each subject comes to study in library?
(a) 2 (b) 4 (c) 6 (d) 8.
2. Number of shelves required to keep books of Moral Education
(a) 2 (b) 3 (c) 5 (d) 7
3. Total number of shelves-required to keep all the books
(a) 17 (b) 15 (c) 12 (d) 10.
4. If cost of each shelf is ` 500, then total cost of all shelf required is Rs
(a) ` 8000 (b) ` 8500 (c) ` 9000 (d) ` 10000.
Solution:
1. Maximum number of students of each subject comes to study in library = HCF
of 12, 18, 30 and 42, that is 6.
2. In one shelf we can keep 6 books, so number of shelves required to keep books
of Moral Education = 42/6 = 7.
3. Total number of books are 12 + 18 + 30 + 42 = 102, hence total number of shelves
required to keep all the books = 102/6 = 17.
4. Total cost of all shelves =17 x 500 = ` 8500.

ANSWERS
1. (c) 2. (d) 3. (a) 4. (b)

Case Study 2
There are 110 students in a school out of these 20 students scored marks more than or
equal to 95% levelled section-A, 25 students scored marks more than or equal to 90% but
less than 95% levelled section-B, 30 students scored marks more than or equal to 85%
but less than 90% levelled section-C and 35 students scored marks more than or equal
to 80% but less than 85% levelled
section-D. School decided to award
all these students with suitable
prizes as per their percentage they
scored. To save the time of award
ceremony students are divided in
the groups of maximum and equal
number of students in each group
and no student of any section can
be grouped with other section.

1. What is the number of students in each group?


(a) 3 (b) 5 (c) 6 (d) 7.
Real Numbers 27

2. Students of section-D will go to receive their prizes in how many groups?


(a) 3 (b) 5 (c) 6 (d) 7.
3. Students of section-B will go to receive their prizes in how many groups?
(a) 3 (b) 5 (c) 6 (d) 7.
4. Total number of groups
(a) 18 (b) 20 (c) 22 (d) 24.
Solution:
1. Total number of students in each group = HCF of 20, 25, 30 and 35 that is 5.
2. Students of section-D will go to receive their prizes in 7 groups.
3. Students of section-B will go to receive their prizes in 5 groups.
4. Total number of groups = 110/5 = 22.

ANSWERS
1. (b) 2. (d) 3. (b) 4. (c)

Case Study 3
We know that the sum, difference, product and division of
two rational numbers are always a rational number and sum,
difference, product and division of one rational number and
other irrational number is always an irrational number.
Answer the following questions:
1. Product of 2 and ÷3 is
(a) rational number (b) irrational number
(c) ÷12 (d) both b and c.
2. Difference of –8 and ÷8 is
(a) rational number (b) irrational number
(c) –8 –2 ÷2 (d) both b and c.
3. Division of ÷12 and ÷3 is
(a) rational number (b) irrational number
(c) 2 (d) both a and c.
4. Sum of 3 + ÷2 and 3 –÷2 is
(a) rational number (b) 6
(c) ÷4 (d) both a and b.
Solution:
1. As 2 is a rational number and ÷3 is an irrational number so their product is an
irrational number which is equal to ÷12 also.
2. As –8 is a rational number and ÷8 is an irrational number so their difference will
be an irrational number which is equal to –8 –2÷2 also.
3. As division of ÷12 and ÷3 is ÷4 = 2, which is a rational number.
4. As sum of 3 +÷2 and 3 –÷2 is 6, which is a rational number.
28 S. Chand’s New Mathematics for Class X

ANSWERS
1. (d) 2. (d) 3. (d) 4. (d)

Case Study 4
A Mathematics Exhibition is being conducted in your School and one of your friends
is making a model of a factor tree. She has some difficulty and asks for your help in
completing a quiz for the audience.
Observe the following factor tree and answer the following: (CBSE Question Bank)
x

5 2783

y 253

11 z

1. What will be the value of x?


(a) 15005 (b) 13915 (c) 56920 (d) 17429
2. What will be the value of y?
(a) 23 (b) 22 (c) 11 (d) 19
3. What will be the value of z?
(a) 22 (b) 23 (c) 17 (d) 19
4. According to Fundamental Theorem of Arithmetic 13915 is a
(a) composite number (b) prime number
(c) neither prime nor composite (d) even number
5. The prime factorisation of 13915 is
(a) 5 × 113 × 133 (b) 5 × 113 × 232
2
(c) 5 × 11 × 13 (d) 5 × 112 × 132

ANSWERS
1. (b) 2. (c) 3. (b) 4. (a) 5. (c)

Competency Based Questions


There are two concentric circular paths of radii 7 metres and 14 metres having centre
Point O, as shown in the given figure.
Two groups of three people each, Ram, Rahim and Nanak started walking from a point
A on the small circular path at 10:00 am and at the same time Krishna, Thomas and Arjun
started walking from point B on the bigger circular path. Points O, A and B are collinear.
Ram takes 3 minutes, Rahim takes 6 minutes and Nanak takes 9 minutes to complete
one round of small Circular Path whereas Krishna, Thomas and Arjun take 4 minutes,
6 minutes and 8 minutes respectively to complete half of the bigger circular path.
Real Numbers 29

O A B

Answer the following questions:


1. At what time Ram, Rahim and Nanak will again meet at point A?
2. How many complete rounds were taken by Ram during this process of meeting
again after they started walking?
3. At what time Krishna, Thomas and Arjun will meet again at point B?
4. If Nanak completes two rounds then in same time how many rounds will be
completed by Ram?
5. How many rounds will be completed by Arjun in the same time in which Thomas
completes 4 rounds?
6. If Rahim takes 8 rounds then in the same time how many rounds will be completed
by Krishna?
7. After they start walking at what time again Ram, Rahim, Nanak will be at point
A and Krishna, Thomas and Arjun will be at point B together?
Solutions:
1. Ram, Rahim and Nanak will again meet at point A at 10:18 am. They will meet
again after 18 minutes which is the calculated LCM of the time taken by them in
completing one round respectively.
2. In 18 minutes Ram will complete 18/3 = 6 rounds.
3. Krishna, Thomas and Arjun will meet again at point B at 10:48 am as time taken
by them to complete their round in 8, 12 and 16 minutes and their LCM is 48.
4. 6 Rounds, (Nanak completes 2 rounds in 18 minutes but in 18 minutes Ram
completes 6 Rounds as he takes only 3 minutes to complete one rounds). as ram
takes only 3 minutes to complete one round.
5. Thomas completes 4 rounds in 48 minutes but Arjun will complete 3 rounds as
he completes one round in 16 minutes.
6. Six rounds Rahim will complete 8 rounds in 48 minutes, so in 48 minutes Krishna
will complete 6 rounds as he completes one round in 8 minutes.
30 S. Chand’s New Mathematics for Class X

7. They all will meet again at Points A and B at 12:24 pm. As LCM of 3, 6, 9, 8, 12
and 16 is 144 minutes which is equal to 2 hours and 24 minutes.
ANSWERS
1. 10 : 18 a.m 2. 6 3. 10 : 48 a.m 4. 6 5. 3
6. 6 7. 12 : 24 p.m

WORKSHEET # 1
FOR PRACTISING THE PROBLEMS BASED ON TRUE/FALSE
Say True (T) or False (F):
1. LCM of two or more numbers is always divisible by their HCF.
2. 7 × 11 × 13 + 13 is a prime number.
3. HCF of two prime numbers is always 1.
n
4. (38) can end with zero.
5. Every real number is always rational.
6. Product of a non-zero rational and an irrational is always an irrational number.
7. Reciprocal of an irrational number is always an irrational number.
8. There are infinitely many rational numbers between any two irrational numbers.
9. Product of two prime numbers is always equal to their LCM.
10. There is no irrational number between any two rational numbers.
11. HCF of two numbers can be 18 if their LCM is 380.
12. LCM of two co-prime numbers is always 1.
ANSWERS
1. T 2. F 3. T 4. F
5. F 6. T 7. T 8. T
9. T 10. F 11. F 12. T

WORKSHEET # 2
FOR PRACTISING THE PROBLEMS BASED ON
FILL IN THE BLANKS

Fill in the Blanks:


1. _____________ is only even prime number.
 HCF of two or distinct natural numbers 
2.   is always a _____________.
 LCM of same set of distinct natural numbers 
n
3. Unit’s place digit of (15) , n ∈ N is always _____________.
4. E
 very composite number can be factorized as a product of primes and this
factorization is _____________.
5. Sum of a rational number with an irrational number is always _____________.
6. For any two positive integers ‘a’ and ‘b’, HCF (a, b) × LCM (a, b) = _____________.
Real Numbers 31

7. Difference of _____________ and _____________ is always an irrational number.


8. _____________ is neither prime nor composite.
9. Every rational number and every irrational number is always _____________.
1 1
10. LCM of , is _____________ and their HCF is _____________.
2 3

ANSWERS
1. 2 2. fractional number 3. 5
4. unique 5. irrational 6. a × b
7. rational and irrational 8. 1 9. real number
1
10. 1 and
6

WORKSHEET # 3
FOR PRACTISING THE PROBLEMS BASED ON
MATCH THE COLUMN

Match the Columns:


Column A Column B
1. Factors of every composite (a) Product of two numbers
number are unique and this
statement is known as
2. Reciprocal of a rational number (b) Fundamental Theorem of Arithmetic
is always
3. LCM of two numbers is always (c) HCF
divisible by their.
4. Product of two consecutive (d) A rational number
integers is always divisible by
5. LCM × HCF of two numbers (e) 2
is equal to

ANSWERS
1. (b) 2. (d) 3. (c) 4. (e) 5. (a)
32 S. Chand’s New Mathematics for Class X

WORKSHEET # 4
FOR PRACTISING THE PROBLEMS BASED ON
ASSERTION REASONING

Directions: The following questions consist of two statements, one labelled the
Assertion (A) and the other labelled as Reason (R). You are to examine these two
statements carefully and decide if the Assertion (A) and the Reason (R) are individually
true and if so, whether the Reason is a correct explanation of the item, Assertion. Select
your answers of these questions using the codes given below and mark your answer
sheet accordingly.
Codes:
(a) Both (A) and (R) are true and (R) is the correct explanation of (A).
(b) Both (A) and (R) are true and (R) is the incorrect explanation of (A).
(c) (A) is true but (R) is false.
(d) (A) is false but (R) is true.
1. Assertion : LCM and HCF of two numbers are 105 and 5. If one of the number
is 15 then other will be 35.
Reason : LCM × HCF = Product of two numbers.
2. Assertion : LCM of two numbers is always a factor of their HCF.
Reason : LCM × HCF = Product of two number
3. Assertion : Factors of every composite number are unique.
Reason : Fundamental Theorem of Arithmetic
4. Assertion : (÷2 + ÷5) is irrational
Reason : 
Sum and difference of two irrational numbers can be rational
number or can be irrational number.
5. Assertion : p is an irrational number.
Reason : 
Ratio of circumference and diameter is always approximate value,
never exact.

ANSWERS
1. (a) 2. (d) 3. (a) 4. (b) 5. (a)

WORKSHEET # 5
FOR PRACTISING THE PROBLEMS BASED on
MCQs WITH MORE THAN ONE CORRECT OPTIONS
1. There is no number between 0.9 and 1
(a) Rational (b) Irrational (c) Real number (d) None of these
n
2. Numbers in the form of (4) , n ∈ N can end with the digit
(a) 5 (b) 4 (c) 1 (d) 6
Real Numbers 33

a
3. If a = ÷12 and b = ÷3 then is a/an
(a) rational number
b (b) irrational number
(c) integer (d) natural number
4. Number ÷3 + ÷2 is not a/an
(a) whole number (b) rational number
(c) integer (d) irrational number
5. Number 2.375 is
(a) rational number (b) irrational number
(c) non-terminating repeating decimal (d) None of these
6. If HCF of two numbers is 1 then two numbers can be
(a) prime (b) coprime (c) cannot say (d) none of these
7. LCM of numbers 1, 2, 3 is equal to their
(a) product (b) division (c) sum (d) None

ANSWERS
1. (a), (b) and (c) 2. (b) and (d) 3. (a), (c) and (d)
4. (a), (b) and (c) 5. (a) and (c) 6. (a) and (b)
7. (a) and (c)

Important Results to Remember


1. If H.C.F. (a, b) = 1, then a and b are said to be relatively primes or co-primes.
2. If a = qb + r, 0 £ r < b then H.C.F. of (a, b) = (b, r).
3. Euclid’s algorithm is a technique of successive division used to find out H.C.F. of
two or more numbers
4. If a, b, m are natural numbers, then H.C.F. of am, bm is m times of H.C.F. of a and
b or for all a, b, m Œ N, H.C.F (am, bm) = m H.C.F. (a, b).
5. H.C.F. of three numbers is the H.C.F. of the H.C.F. of any two of them and the third
numbers.
6. If H.C.F. of (a, b) = 1 and a is a factor of bc then a is a factor of c. This is known as
Gauss’s theorem.
7. The product of L.C.M. and H.C.F. of two numbers a and b is equal to the product
of two numbers.
8. L.C.M. of two prime numbers or of two co-primes is equal to their product.
a
9. A rational number in lowest terms has a terminating decimal expression if and
b
only if b has no prime factors other than 2 and 5.
34 S. Chand’s New Mathematics for Class X

a
10. A number which cannot be expressed in the form of , a, b Œ I, b ≠ 0 is known
b
as an irrational number.
1
11. If a be an irrational number, then – a and are also irrational.
a
12. If ab be an irrational number then a + b is also irrational.

13. The decimal representation of an irrational number is always non-terminating,


non-repeating.

14. If a be an irrational number and r a non-zero rational number, then the sum,
difference, product and quotient of r and a are also irrational.
L.C.M. of Numerators
15. L.C.M. of fractions or Rational numbers =
H.C.F. of Denominators
H.C.F. of Numerators
16. H.C.F. of fractions or Rational numbers =
L.C.M.of Denominators

You might also like